Codeforces Round #654 (Div. 2) Editorial

Revision en4, by physics0523, 2020-12-17 20:16:00
Tutorial is loading...

Jury solution: 85699387

Tutorial is loading...
Jury solution: 85699884
Tutorial is loading...
Jury solution: 85699939

UPD: $$$m<\min(a,b)$$$ is wrong, the right text is $$$m\le \min(a,b)$$$

Tutorial is loading...
Jury solution: 85700178
Tutorial is loading...
Jury solution: 85700334
Tutorial is loading...
Jury solution (Solution 1): 85700515

Jury solution (Solution 2): 85700518

Tutorial is loading...
Jury solution: 85700669

Feel free to share your approach here!

Tags 654, #tutorial

History

 
 
 
 
Revisions
 
 
  Rev. Lang. By When Δ Comment
en4 English physics0523 2020-12-17 20:16:00 0 (published)
en3 English physics0523 2020-12-17 15:55:36 67 Tiny change: '5699939]\n[tutoria' -> '5699939]\nUPD: $m<$\n[tutoria' (saved to drafts)
en2 English physics0523 2020-07-01 19:52:19 54 fixed the link
en1 English physics0523 2020-07-01 19:23:17 540 Initial revision (published)